Zollege is here for to help you!!
Need Counselling
GMAT logo

If the Proposed Tax Reduction Package is Adopted this year, the Library GMAT Critical Reasoning

Overview es 2Overview en 2RegistrationExam PatternPreparation TipsPractice PaperResultCut offmock testNews
Sayantani Barman's profile photo

Sayantani Barman

Experta en el extranjero | Updated On - Feb 23, 2023

Question: If the proposed tax reduction package is adopted this year, the library will be forced to discontinue its daily story hours for children. But if the daily story hours are discontinued, many parents will be greatly inconvenienced. So the proposed tax reduction package will not be adopted this year.

Which one of the following, if assumed, allows the argument’s conclusion to be properly drawn?

(A) Any tax reduction package that will not force the library to discontinue daily story hours will be adopted this year.
(B) Every tax reduction package that would force the library to discontinue daily story hours would greatly inconvenience parents.
(C) No tax reduction package that would greatly inconvenience parents would fail to force the library to discontinue daily story hours.
(D) No tax reduction package that would greatly inconvenience parents will be adopted this year.
(E) Any tax reduction package that will not greatly inconvenience parents will be adopted this year.

Correct Answer: A
Explanation:

The GMAT's critical reasoning portion will rate your analytical and critical thinking skills. To reply clearly, one needs a lot of mental power.
Before making a choice, each alternative must be carefully evaluated. There will only be one accurate answer out of the possible five. To select the best choice, the candidate needs to take into account a number of variables.
Let's approach the answer choices.

A: Incorrect

It is an incorrect answer. We will be compelled to stop hosting daily story hours due to the planned package. The rule in (A) is useless to us because it doesn't apply to the suggested package that we are discussing in the argument. As it doesn't relate to the suggested package we are discussing, it leads nowhere else.
If the question had read, "Any tax reduction package that will compel the library to stop offering daily story hours will not be adopted this year," then the response (A) would have been appropriate.

B: Incorrect

It is the wrong choice. If it were established that this particular tax scheme does not inconvenience parents, but we are told that it does. The conditional chain could provide the assumption we're searching for.
The diagram is as follows:
A tax on parents who are disadvantaged was passed. (chain is the opposite of this)
inconvenience for parents. (The stimulant informs us of this)
As a result, because the conditions don't fit, we cannot deduce that the chain's tax was implemented.

C: Incorrect

It is an incorrect answer. If it were established that this particular tax scheme does not inconvenience parents, but we are told that it does. The conditional chain could provide the assumption we're searching for.
The diagram is as follows:
A tax on parents who are disadvantaged was passed. (chain is the opposite of this)
inconvenience for parents. (The stimulant informs us of this)
As a result, because the conditions don't fit, we cannot deduce that the chain's tax was implemented.

D: Correct

It is the correct choice. This is the concluding passage.

E: Incorrect

It is an incorrect choice. E appears to be saying: Will be Adopted -> Not Inconvenienced.

The stimulus's premises result in "Inconvinced" for us. You cannot transition from this to "Will Not Adopted" under the aforementioned criterion.

Because, once again, you need to get from the point when parents are inconvenienced to the tax plan not being adopted, the contrapositive, Not Adopted -> Inconvenienced, doesn't help us either. The foregoing contrapositive only allows us to proceed as far as the parents' inconvenience. That doesn't benefit us because we learn this truth from the premise that this rule is obsolete.

“If the proposed tax reduction package is adopted this year, the librar” – is a GMAT Critical question. To answer the question, a candidate can either find a piece of evidence that would weaken the argument or have logical flaws in the argument. GMAT critical reasoning tests the logical and analytical skills of the candidates. This topic requires candidates to find the argument's strengths and weaknesses or the logical flaw in the argument. The GMAT CR section contains 10 -13 GMAT critical reasoning questions out of 36 GMAT verbal questions.

Suggested GMAT Critical Reasoning Samples

*The article might have information for the previous academic years, please refer the official website of the exam.

Ask your question

Subscribe To Our News Letter

Get Latest Notification Of Colleges, Exams and News

© 2024 Zollege Internet Private Limited